GMAT Error Log 2017- Quant and Verbal - Best Questions.pdf

March 27, 2018 | Author: Gizem Koc | Category: Cartesian Coordinate System, Life Expectancy, Triangle, Prime Number, Symmetry


Comments



Description

R # Source Ty Question CommentsOA Take Aways * Percent change = Difference / Original * %100 * Check percentages, the sum of bars may not be 100. Then calculate as 25 of 75 ; then what of 100 for the percentage questions. 2 Econ-Test IR The number of participants who reported irritability was about 32% greater in ????? than in Group D. * READ Marine biologist: While we have known that larger marine animals such as whales and dolphins communicate using sound, the fact that smaller marine animals also use sound is a relatively new discovery. These low-frequency sounds produced by animals in the mesopelagic zone--that area between 200 and 1,000 meters below the ocean surface--may be evidence that these smaller animals can commu You know that it is necessary to find the assumption first, as the fact is 7 Econ-Test IR Indicate two different statements as follows: one statement identifies an assumption required by the marine biologist's argument and the other identifies a possible fact that, if true, would provide significant logical support for the required assumption. Make only two selections, one in each column. dependent on it. Preconceived notions of the world, especially that of the so-called Third World, have colored our perceptions enough to blind us to its true state, illustrated by decades of reliable statistics. The currently prevalent and popular juxtaposition of the struggling Third World and the flourishing First World is consistent with the state of the world in the mid - twentieth century, not the world in which we curren With which of the following statements about life expectancy would the author of the passage most likely agree? A) Life expectancy is inversely proportional to fertility rate. B) Having a high life expectancy should be a requirement for common consideration as a "First World" country. C) Life expectancy is an indicator of a country's overall state. C) Very good! In order for the author to use a statistic such as the average life expectancy in a country, that statistic must show something about the country as a whole or else the information would be meaningless. D) Although life expectancy is currently high in most countries, it will not remain so in those countries commonly referred to as "Third World" countries. Econ-Test RC E) Low fertility rates cause life expectancy to rise. A) This answer choice incorrectly presumes that there is a relationship between the two factors. However, even in the case the author C presents, these factors are not necessarily related. The passage is primarily concerned with A. presenting various groups’ views of the motives of those proposing certain legislation B. contrasting the reasoning of various groups concerning their positions on certain proposed legislation Evaluate : to judge or calculate the quality, importance, amount, or value of C. tracing the process whereby certain proposed legislation was eventually enacted something 20 D. assessing the success of tactics adopted by various groups with respect to certain proposed legislation D (OG 89) OG16 verbal1 RC E. evaluating the arguments of various groups concerning certain proposed legislation B Author only states the position of opponenets does not mean evaluation. 16 Diag.Q. PS E If there is a long equation, try to get what is asked exactly. We see that we can select a boy in 3 ways (since there are 3 boys) and we can select a girl in 2 ways (since there are 2 girls). So we can make a pair in 3*2 = 6 ways. The basic counting principle deals with problems having ‘distinct spots’ and ‘available contenders’. Here we have 1 spot for a boy and 1 spot for a girl i.e. 2 distinct spots. There are 3 contenders for the empty ‘boy spot’ and 2 contenders .com/blog/2011/10/quarter-wit-quarter-wisdom-the-dreaded-combinatorics/ Veritas Blog PS There are 3 boys and 2 girls. We want to select a pair of one boy and one girl for a dance. In how many ways can we do it? Also notice here that it is not 3+2 = 5 ways. This is so because we have to choose a boy AND a girl simultaneously. For every boy, we6 could choose a girl in 2 ways and there are 3 boys so we can choose a pair in 3*2 ways. If we had to cho 2 – no, it’s not even | 3 – no, its digits sum to 4 | 5 – no, it doesn’t end in 0 or 5 7 – here’s where we may need to get creative. We could perform the long division, but we could also do as we did above with 7 and 11 and find “easy to calculate” multiples of 7 and work from there. We should know that 2100 is a mult 17 – 1700 is a multiple of 17; add 340, another multiple, to get closer to 2011 and you have 2040. Subtract 17 and that’s 2023, and subtractIf17 youagain keep toinget mind thatthat range you(2023 only –need 17 =to2006) test primes; and recognize that you that only 17have is nottoa test factor of 2011. | 19 23 – 2300 is a multiple of 23. Subtract 230 to get 2070. Subtract 69 to get 2001, add 23 to bracket the range at 2024 and you can tell that 23up is to notthe a factor squareofroot 2011.of |the 29next – 2900 highest is a multiple square, and of 29. that Subtract you can290 playtothe get 2610; do it again to g 37 – 3700 is a multiple of 37, but far too large, so divide by 2 to get closer to the intended range: 1850. Add 185 to get 2035. Subtract 37 to multiples establish the game range instead around of performing 2011 and you longhave division, 1998,prime so 37number is not a testing factor ofcan2011. be Let’s top and Man-CAT1 PS Is 2011 prime? 41: 4100 is a multiple of 41, and if we cut that in half we’re in the range at 2050. Subtract 41 to get 2009 and you can realize that 41 won’t be done a factor muchofmore2011.efficiently | 43:than4300youis amight multiple think! of 43, and if we cut that in half we’re close at 215 Right triangle PQR is to be constructed in the xy-plane so that the right angle is at P and PR is parallel to the x-axis. The x and y coordinates of P, Q and R are to be integers Consider that satisfy the green the inequalities dotted rectangle. -4<x<5 and The6<y<16. x coordinate How many varies different from -4 totriangles 5 and the with y coordinate these properties varies can frombe6 constructed? to 16. PR has to be parallel to x axis. Since it is a right angled triangle, PQ will be parallel to y axis. (A) 110 Now let’s first fix the vertex P. In how many ways can you choose the coordinates of P? The x coordinate of P can vary from -4 to 5 i.e. it can take 10 values. The y coordinate of P can vary from 6 to 16 i.e. it can take 11 values. In all, the c (B) 1100 Once vertex P is fixed, the x coordinate of Q will be the same as the x coordinate of P (since PQ is parallel to y axis) and the y coordinate of R will be the same as the y coordinate of P (since PR is parallel to x axis). (C) 9900 The y coordinate of Q can be chosen in 10 ways. (Out of the 11 values of y coordinate, one is occupied by P so 10 are leftover.) (D) 10000 The x coordinate of R can be chosen in 9 ways. (Out of 10 values of x coordinate, one is occupied by P so 9 are leftover.) p.com/blog/2011/09/quarter-wit-quarter-wisdom-use-variations-to-succeed/ Veritas Blog PS (E) 12100 Total number of ways of making the triangle = 110 * 10 * 9 = 9900 C How many triangles with positive area can be drawn on the coordinate plane such that the vertices have integer coordinates (x,y) satisfying 2 < x < 4 and 5 < y < 7? (A) 72 (B) 76 (C) 78 (D) 80 p.com/blog/2011/09/quarter-wit-quarter-wisdom-use-variations-to-succeed/ Man-CAT1 PS (E) 84 https://www.veritasprep.com/blog/2011/09/quarter-wit-quarter-wisdom-use-variations-to-succeed/ B Now that we understand this process, let’s examine the symmetry in this. Say we flip the image along the vertical axis – what do we get? The figure is still exactly the same, but now the order of cells is reversed to be 4, 3, 2, 1. The pathw To reach Cell 1, the ball needs to turn left-left-left. To reach Cell 4, the ball needs to turn right-right-right. Since the probability of turning left or right is the same, the situations are symmetrical. This will be the same case for Cells 2 and 3. Therefore, Approach: by symmetry, Using symetry we see that: in symetrical The probability possibility of reaching questions Cell 1 = the probability of reaching Cell Similarly: The probability of reaching Cell 2 = the probability of reaching Cell 3. (There will be multiple ways to reach Cell 2, but the ways of reaching Cell 3 will be similar, too.) The total probability = the probability of reaching Cell 1 + the probability of reaching Cell 2 + the probability of reaching Cell 3 + the probability Wayoftoreaching calculateCell all4possible =1 outcomes is : Because we know the probability of reaching Cells 1 and 4 are the same, and the probabilities of reaching Cells 2 and 3 are the same, this equation (possiblecan outcome be written of anas:event)" 2*(the(number probability of events) of reaching Cell 1) + 2*(the probability of reaching C Let’s find the probability of reaching Cell 1: After the first opening (not the peg, but the opening between pegs 1 and 2 in the first row), the there ball moves are 3 left rows(between of pegs to pegs pass, 1 and each2 row in second gives 2row) ways. or Thus, right (between all possible pegs 2 and 3 in second r um/the-figure-shown-represents-a-board-with-4-rows-of-pegs-and-36668.html PS After that, the ball must move left again – the probability of this occurring is also 1/2, since probability of moving left or right is equal. Finally, outcomes the ball =must 2*2*2 turn= 2^3=8 left again to reach Cell 1 – the probability of this occurring is, again, 1/2. Th If a rectangle with length units and width units is inscribed in a circle of radius 5 units, what is the value of a + b? 100 if it is too easy to be true, check again the Mexican Senate recently voted to expand public school sex education programs to kindergarten.” This statement indicates that public school sex education programs already exist; the programs cannot be expanded to include a spec 14 Manhattan CAT RC D Expand !! Premise: There is overcrowding in the schools (not just high schools). Conclusion: There has been a decline in the reading skills among high school students. Assumption: The overcrowding in the schools (in general) has caused the declining reading skills among high school students. The correct answer will break the connection between overcrowding in the schools and declining reading skills among high school students. Eliminate answers that alter the scope. B is too broad. The argument is only about high school reading skills. In B, we Reading skills among high school students in Gotham have been steadily declining, which can only be the result of overcrowding in the schools. don't know which schools have the lower reading scores. The scores could be Which of the following, if true, most seriously weakens the argument expressed above? lower because of the students in schools other than high school. Eliminate B. D reverses the scope. Whereas the argument refers to reading skills among high a) The high school system in Gotham succeeds in giving students a good education at considerably less cost than do most systems. school students, D refers to reading skills in general; whereas the argument b) Several cities have found that overcrowding in the schools is not always associated with lower reading scores. refers to overcrowding in general -- not just in high school -- D refers to c) Gotham schools have a greater teacher-to-students ratio than most other school systems. overcrowding only in high schools. Eliminate D. d) Students' reading skills have not declined in other cities where the high schools are just as crowded as those of Gotham. Only answer choice E breaks the connection between overcrowding in general http://www.beatthegmat.com/declining-reading-score-t81954.html CR e) Schools are not overcrowded in many cities where high school reading scores have declined more than they have in Gotham. E and poor reading skills in high school. Recent advances in surgical procedures have not decreased the average time for recovery from medical operations. Compared to ten years ago, the average post-operative hospital stay for patients undergoing surgery at the ten largest hospitals has actually increased by four days. Although mortality rates for operative procedures are much lower, patients are requiring more time to recover from these p All of the following, if true, weaken the argument above EXCEPT: B)Fewer than thirty percent of beds in the largest hospitals are occupied by patients recovering from surgery. A)Operative and post-operative mortality rates are the most accurate indicators of quality of medical care. -- This choice doesn't weaken or strengthen the conclusion, as the percentage of occupancy does not tell us anything about avg time of post surgical recovery. Author's reasoning is focused around decline in average time of recovery. Jus B)Fewer than thirty percent of beds in the largest hospitals are occupied by patients recovering from surgery. C)In the past ten years, innovations have allowed previously inoperable conditions to be treated successfully by major surgery.-- This weakens author's conclusion. C)In the past ten years, innovations have allowed previously inoperable conditions to be treated successfully by major surgery. D)Every year, many surgical procedures that previously required hospital stays are simplified enough that they can safely be performed in outpatient clinics. D)Every year, many surgical procedures that previously required hospital stays are simplified enough that they can safely be performed in outpatient clinics. -- This weakens author's conclusion and implies that hospital stay would reduce as the surgeries can be performed in clinics. ub.com/forum/recent-advances-in-surgical-procedures-have-not-decreased-145430.html CR E)Average surgical recovery time, measured by hospital stay plus time spent disabled from normal activities at home, has decreased by twelve percent in the last ten years. E)Average surgical recovery time, measured by hospital stay plus time spent disabled from normal activities at home, has decreased B by twelve percent in the last ten years.-- This also clearly weakens author's conclusion Meteor showers or storms left in the wake of disintegrating comets fall towards the earth and have caused several accidents when they collide with unmanned communication satellites. Irish astronomer James Lawrence postulated a method for calculating the position of the meteor dust at Earth's orbit by studying the dust ejected in 1866 by comet 55P/ Tempel-Tuttle and the Leonid shower return of 1 Which of the following, if true, most seriously weakens the argument above? 1)Predictions about the position of approaching meteor showers are much more accurate today than they were in the last decade of the nineteenth century. Choice C. Evidence other than that used by Lawrence has previously enabled scientists to predict the position of the meteor dust as it approaches the earth. 2)Scientists have corroborated Lawrence’s work and established the significance of the Leonid shower to calculations of the position of meteor dust. Premise: This method allowed him to predict with reasonable accuracy the position of the meteor dust as it approaches the earth. 3)Evidence other than that used by Lawrence has previously enabled scientists to predict the position of the meteor dust as it approaches the earth. Conclusion: accidents caused by the collision of meteor showers with unmanned communication satellites will be prevented 4)Scientists have not been able to understand exactly how the meteor showers disintegrate before they reach the earth, yet manage to have a significant impact on the unmanned satellites. CR 5)Some scientists have found several discrepancies between Lawrence’s work with meteors and his earlier work. If accidents caused by the collision of meteor showers with unmanned communication satellites will be prevented because this method c allowed him to predict with reasonable accuracy the position of the meteor dust as it approaches th In Colorado subalpine meadows, nonnative dandelions co-occur with a native flower, the larkspur. Bumblebees visit both species, creating the potential for interactions between the two species with respect to pollination. In a recent study, researchers selected 16 plots containing both species; all dandelions were removed from eight plots; the remaining eight control plots were left undisturbed. The con The scientists concluded that "the presence of dandelions facilitates pollination (and hence seed production) in the native species by attracting more pollinators to the mixed plots." We want to weaken this conclusion. Which of the following, if true, most seriously undermines the researchers’ reasoning? (A) If all the bees were visiting dandelions, then this would not explain why the larkspurs got so pollinated, producing a great quantity of seeds. This is simply inconsistent with the evidence. This is not correct. (B)This is a strengthener. If this is true, then it would explain why having dandelions in the plot would result in more pollination for the larkspurs. This is a typical GMAT CR trap, having a strengthener for a weakener, or vice versa. This is (A) Bumblebees preferentially visit dandelions over larkspurs in mixed plots. (C) This is a problems down the road, a reason why in the big picture dandelions might be a problem for larkspars, but it doesn't do anything to address the issue of which plants get pollinated and how many seeds are produced. This is n (B) In mixed plots, pollinators can transfer pollen from one species to another to augment seed production. (D) Even if this is true, it is not relevant, because in terms of the experiment, only seed production was measured. There was no measurement of seed germination: at most, it was merely inferred from seed production. The experiment l (C) If left unchecked, nonnative species like dandelions quickly crowd out native species. (E) This may appear irrelevant at first glance, but think about it. The control plot (with dandelions) produced more seeds than the experimental, dandelion-free plot. How was that latter plot prepared? The prompt says: "all dandelions w (D) Seed germination is a more reliable measure of a species’ fitness than seed production. OG CR (E) Soil disturbances can result in fewer blooms, and hence lower seed production. (E) has to be the OA. This is a brilliant question, because at first glance, it may appear that (E) is entirely out-of-scope and irrelevant.e You have to think about the details of the prompt to recognize why it is so relevant. Due to an easing of state sanctions against hunting, Deersdale Preserve has experienced an influx of hunters in the last several months. During this time, the silvertail fox, a popular target for hunters, has seen a marked decrease in population. Therefore, to prevent the population of silvertail fox—an important apex predator responsible for keeping the Deersdale County’s rabbit population in check—fro Which of the following, if true, may suggest that stricter sanctions against hunting would not have the desired effect? A.The population of rabbits has surged ever since the hunting sanctions in Deersdale County were lifted. B.The silvertail fox population varies greatly throughout the year, especially during winter when prey becomes scarce. C.The local authorities are expecting even more hunters in the coming year to arrive to the park. (B) would be true in normal circumstances. That's part of the point. It's nothing out of the ordinary cycle of seasons. Lele wrote this question so that most users would pass over all five answers on the first go, and then have to come back D.The silvertail fox had been experiencing a population surge shortly before the state sanctions against hunting were eased. Magosh CR E.The grey wolf, a large predator that competes with the silvertail fox over Deersdale Preserve’s rabbit population, has seen its numbers decrease since the arrival of the We hunters. need something, anything, that would suggest that a decline in the silvertail fox is not due solely to the increased hunting. Well,b winter also reduces the fox population. Is the question happening in winter, or is winter imminent? Har An independent analyst asserts that the new Pokia digital camera for mobile phones is more precise, of higher quality and costs less than any of the popular competing models. As a result, Pokia will become the more desirable, low-priced alternative to currently existing models of digital cameras. Which of the following, if true, would most challenge the argument above? (A) Many retailers already carry one or more low-priced digital cameras and are disinclined to carry another. (B) Several lower-priced models of digital cameras will soon be introduced by other mobile phone manufacturers. (C) The Pokia Corporation’s digital camera can be used in conjunction with higher-priced mobile phones manufactured by other companies. This is a weaken question. The analyst asserts that it is more precise, of higher quality, and less expensive than other options. Well cost and precision are not very subjective, but quality certainly is. If, as in answer choice E, the analysts' o (D) Most of the individuals and companies that would be expected to make up the potential market for the Pokia digital camera have already fulfilled their digital camera needs. http://gmatclub.com/forum/pokia-digital-camera-163310.html CR (E) The independent analyst whose views were incorporated in the statement above has used measures of quality that are not universally accepted by the consumer public. Answer choices A and D are both incorrect because they each focus only on the practical success of the camera, and not on whether e or not it is a preferred alternative. If many retailers refuse to carry the camera, as in A, then sure that m The recent decline in the value of the dollar was triggered by a prediction of slower economic growth in the coming year. But that prediction would not have adversely affected the dollar had it not been for the government’s huge budge Which of the following, if true, would most seriously weaken the conclusion about how to prevent future currency declines? Which argument we are trying to weaken here : government’s huge budget deficit, which must therefore be decreased to prevent future currency declines. The recent decline in the value of the dollar was triggered by a prediction of slower economic growth in the coming year. But that prediction would not have adversely affected (A) Thethe government dollar hadhas it not made been little forattempt the government’s to reduce the hugebudget budgetdeficit. deficit, (This which does must nottherefore provide evidence be decreased that budget to prevent deficit future was currency the only reason) declines. Which of the following, if true, would most seriously weaken the conclusion about how to prevent future currency declines? (B) The budget deficit has not caused a slowdown in economic growth.(Not relevant in the argument here) (A) The government has made little attempt to reduce the budget deficit. (C) The value of the dollar declined several times in the year prior to the recent prediction of slower economic growth. (This does not talk about the budget deficit was present at these times of decline or not) (B) The budget deficit has not caused a slowdown in economic growth. (C) The value of the dollar declined several times in the year prior to the recent prediction of slower economic growth. (D) Before there was a large budget deficit, predictions of slower economic growth frequently caused declines in the dollar’s value. (This directly concludes that large budget deficit is not responsible for the decline in the dollar's value w (D) Before there was a large budget deficit, predictions of slower economic growth frequently caused declines in the dollar’s value. lub.com/forum/the-recent-decline-in-the-value-of-the-dollar-was-triggered-130234.html CR (E) When there is a large budget deficit, other events in addition to predictions of slower economic growth sometimes trigger declines in currency value. (E) When there is a large budget deficit, other events in addition to predictions of slower economic growth sometimes trigger declines d in currency value. (This infers that " government’s huge budget deficit, which must therefore be decr The price the government pays for standard weapons purchased from military contractors is determined by a pricing method called “historical costing.” Historical costing allows contractors to protect their profits by adding a percentage increase, based on the current rate of inflation, to the previous year’s contractual price. Which of the following statements, if true, is the best basis for a criticism of historical costing as an economically sound pricing method for military contracts? (A) The government might continue to pay for past inefficient use of funds. (A) The government might continue to pay for past inefficient use of funds. (B) The rate of inflation has varied considerably over the past twenty years. This correctly shows that bad deals in the past will continue to affect contracts in the future. Pick A. (C) The contractual price will be greatly affected by the cost of materials used for the products. (D) Many taxpayers question the amount of money the government spends on military contracts. (B) The rate of inflation has varied considerably over the past twenty years. CR GMAT Official Guide for (E) The pricing Verbal method2nd Review, based on historical costing might not encourage the development of innovative weapons. Edition if the rate of inflation varies, it will still increase the cost either by a little or alot. this doesnt say anything about historical prices. a A system-wide county school anti-smoking education program was instituted last year. The program was clearly a success. Last year, the incidence of students smoking on school premises decreased by over 70 percent. Which of the following, if true, would most seriously weaken the argument in the passage? (A) The author of this statement is a school system official hoping to generate good publicity for the anti-smoking program. (B) Most students who smoke stopped smoking on school premises last year continued to smoke when away from school. (B) Most students who smoke stopped smoking on school premises last year continued to smoke when away from school. Incorrect - this statement is a bit neutral. It neither strengthens the argument nor necessarily weakens it. Furthermore, if the ultimate goal of the program was to decrease on school incidences, then it was still a success. (C) Last year, another policy change made it much easier for students to leave and return to school grounds during the school day. (D) The school system spent more on anti-smoking education programs last year than it did in all previous years. (C) Last year, another policy change made it much easier for students to leave and return to school grounds during the school day. CR (E) The amount of time students spent in anti-smoking education programs last year resulted in a reduction of in-class hours devoted to academic subjects. Correct - We needed something that indicated another motivation outside of the program. This fulfills that necessity and weakens the c argument. If we can positively associate the decrease in on-school incidences to another policy, then Degler does more than merely summarizing existing research; he constructs a coherent picture of two centuries of studies dealing with the changing roles of women. (A) Degler does more than merely summarizing (A) Degler does more than merely summarizing…….summarizing and constructs do not go // (B) Degler’s study is more than a mere summarizing of (B) Degler’s study is more than a mere summarizing of ---- summarizing of and constructs do not go well; one is a gerund and the other is a verb (C) Degler has done more than a mere summarizing of (C) Degler has done more than a mere summarizing of ----- same as B he does summirize <> he constructs (D) Degler’s study has done more than summarize merely (D) Degler’s study has done more than summarize merely- comparing a study with he – a person; unparallel ile aynı değil SC (E) Degler does more than merely summarize (E) Degler does more than merely summarize ------- Degler tallies with He, does summarize tallies with constructs --- correct e he does summarizing <> he constructs As accelerating climate change and other man-made environmental degradations create growing alarm across the planet, the Sami people have much to teach the world about how to adapt, survive, and thrive. First, we split the sentence in terms of meaning. What creates growing alarm across the planet? Only the man-made environmental degradations or both climate change and them? The intended meaning should be "the climate change a A. As accelerating climate change B. As accelerated climate change Between A and B, should we use V+ing or V+ed form. To come up with an answer, let think: climate change accelerates by itself or It is accelerated? Clearly, it accelerates itself ==> V+ing form is correct. C. As climate change accelerates D. As climate change is accelerated a) The climates changes accelerated by the soil erosion has become a havoc for the farmers of the Indus valley. -> What caused the acceleration in climate changes = soil erosion SC E. As climate changes are accelerated b) The accelerating car hit the truck before plunging into the valley. -> There is no mention in the sentence who caused the acceleration, e since the car was accelerating by itself and the word accelerating just adds additional information. A.hardy, disease-resistant and yields Wrong. Because "hardy" || disease-resistant, so there should be "and" between "hardy" and "disease-resistant" to make the parallel structure clear. Agricultural scientists continue to search for a strain of wheat that is hardy, disease-resistant and yields more grain per acre. B.hardy, resists disease, and that yields Wrong. If "that is"|| resists || yields ==> the second "that" is wrong. The correct structure should be: a strain of wheat that is hardy, resists disease and yields..... A.hardy, disease-resistant and yields C.hardy and disease-resistant and that yields Correct. "hardy" || "disease-resistant; B.hardy, resists disease, and that yields "that is ...." || "that yields D.hardy and resist disease, yielding C.hardy and disease-resistant and that yields Wrong. "a strain" is singular ==> "resist" is wrong. Should be "resists". E.hardy, disease-resistant, and yielding D.hardy and resist disease, yielding Wrong. Although the structure is parallel, the meaning is wrong. "Hardy" and "disease-resistant" are characteristics of the strain of wheat, but "yielding" is the advantage of the grains, so we cannot put them together in a parallel structu - My professor who is tall, black-hair and who teaches math is very smart. Agricultural scientists continue to search for a strain of wheat that is hardy and SC E.hardy, disease-resistant, and yielding - We can put "tall" and "black-hair" in one parallel structure, but cannot put "tall" + "black-hair" + "teach math". They are in different c categories. resists disease, yielding more grain per acre. >>>> RIGHT Sometimes the members sit at tables with 3 members at one table and 4 members at each of the other tables. Note that one of the errors is assuming that no third element caused both the stated cause and the stated effect. this statement is insufficient.com/forum/a-certain-square-is-to-be-drawn-on-a-coordinate-plane-127018. D Remaining numbers to fill last two digits (3.this is NOT. One of the vertices must be on the origin. The reasoning in the argument is most vulnerable to criticism on the grounds that the argument ______________ The argument contains a causal conclusion that asserts that good wealth is primarily caused by informed career and business choices (education) the conclusion is that research supports the view that gaining wealth is largely the result o A. it's results are out. 211 Only odd factorials: 0!=1 and 1!=1. or 7. 3| x 2 – 4| must be greater than or equal to 0. Many of our adult customers do not appear comfortable with this kind of crowd and some of them have told me that they will no longer stop here for a coffee drink. is a perfect square).8. Or consider the following: individual how can the probability probability that both of each Event AA happening? So.html Gmat Club PS E. We can.3? how can the probability that both Event A and Event B will happen be more than (1) Probability that AA will happen is 0. The following day the buses do the return trip at the same constant speed. the revenue from sales would be 10% greater than that collected last month.71 (2) Probability that B will NOT happen is 0. 5 C. then the probability that both occur. 110/20 E. 96 D. or its officers guilty (C) suggesting that the unusually large contributions to its accounts had been derived from government kickbacks. A bus leaves city M and travels to city N at a constant speed. the probability that he will be able to find the Probability of choosing right numbers in two places = 1/5 * 1/5 = 1/25 correct number in at most 2 attempts is closest to which of the following ? Probability of not choosing right numbers in two places = 1-1/25 = 24/25 -- A. and sometimes they sit at tables with 3 members at one table and 5 members at each of the other tables. presumes. The same here: since P(B)=1−0. So don't need perfect tense here. then 1−y≤01−y≤0. not the veracity of the owner’s e The store manager responds to the owner by _____ (B) The implementation of the owner’s directive is not mentioned by the manager and is not relevant.. what percent less would last month's revenue be compared to this month's revenue? A. Right . There is no mention of poor family background. without providing justification. Also notice that 0≤P(A)≤1 and 0≤P(B)≤1 If Event AA and Event BB are independent.6. (1) |x−2|<2−y| GMAT Club Tests DS (2) x+y−3=|1−y| (2) x+y−3=|1−y|x+y−3=|1−y|. 72 C.. how many different ways can this square be drawn? 4 6 8 10 http://gmatclub. 3. note that the absolute value expression | x 2 – 4| must be greater than or equal to 0. For example 16=4216=42. the absolute value expression | x 2 – 4| must be greater than or equal to 0. 0 is divisible by every integer except 0 (0/5 OK . 6. so the least value of LHS is zero. 2.71=0. 2) Right . so P(A and B)=P(A)∗P(B)P(A and B)=P(A)∗P(B). Not sufficient.25. 2/625 1) = 24/25 * 1/25 = 24/625 C.. The parallelism error : that has suggested blah blah. just that making good choices generally results in gaining wealth. we must consider both the positive and negative values of the absolute value expression: When there is aboslute value on left side. After driving for 2 hours they meet at point P. 151 The factorial of a negative number is undefined. However. to If y2. is the probability that both Event AA and Event BB will happen greater than 0. -The author specifically says that the effect is largely the result of the cause. If on a randomly chosen day in June the sum of all deposits up to and including that day is a prime integer greater than 100. One bus is delayed 24 minutes and the other leaves 36 minutes earlier. how many members will be at the table that has fewer than 6 members? (A) 1 (B) 2 (C) 3 (D) 4 m/club-x-has-more-than-10-but-fewer-than-40-members-sometimes-143987. then P(A and B)=P(A)∗P(B)≤0. . the unusually nor itslarge officers contributions guilty of improper to its accounts relations hadwith beenindustry derivedregulators.html D READ CAREFULLY If for any positive integer x. with its officers guilty E)Parallelism Error: (D) to suggest that the unusually large contributions to its accounts are derived from government kickbacks or that its officers are guilty 1)to suggest that the unusually large contributions to its accounts had been derived from government kickbacks. D which gives x=2=primex=2=prime. (1) When xx is divided by 12 the remainder is 6. even if one does not make an informed career and business choices.html B WHITE and RED (1) INSUFFICIENT: Since this equation contains two variables. but the answer suggests B. overlooks the possibility that people who make informed career and business choices may nonetheless belong to a poor family. he makes a claim as to the relative revenue generated by the average member of each group.25. thenthe y >two 2. (A) questioning the veracity of owner’s evidence (C) CORRECT. Now. at the same time another bus leaves city N and travels to city M at the same constant speed. we have that: |x−2|<1|x−2|<1. 7. So.3. i. the manager is presenting new evidence that implies that the curren (B) arguing that it would be difficult to implement the owner’s directive (C) offering new evidence implying that the status quo is not incompatible with the owner’s goal (D) The manager only states that the average teenager spends as much as the average adult. you can also rule out A as at least 2 Owner of JavaJoint: Over the past year. is the remainder when a is divided by 11 less than the remainder when b is divided by 11? (1) The remainder when aa is divided by 69 is the fifth power of a prime number.com/forum/m21-184291. orum/2-the-federal-investigators-at-stapleton-industries-have-154221. store manager makes no attempt to question the veracity of the statements of the owner. The left hand side of the inequality is an absolute value.html Magoosh SC (E) to suggest that the unusually large contributions to its accounts had been derived from government kickbacks.html 12 Q-26Manhattan CAT2 PS As for your question I doubt that this is a realistic GMAT question. 25/625 orum/a-telephone-number-contains-10-digit-including-a-3-digit-59697. since 2q+12q+1 is an odd number then the power of 2 in xx will be odd (1). and c are consecutive even integers GMAT Club Tests DS (2) ac<0 http://gmatclub. Next. If Joan takes 2 random marbles from the jar.html D (1) |x−2|<2−y|x−2|<2−y.html Gmat Club PS (E) 5 E Focus on any one of the two elements.9): Total 5 A telephone number contains 10 digit. 1. For example: 36=22∗3236=22∗32.com/forum/m27-184484. 48 B. 1/625 At most two attempts: 1) Wrong-1st Attempt. then P(A and B)=P(A)∗P(B)≤0. A jar contains 8 red marbles and y white marbles. however. @ can be 0 or 9 (note that zero is divisible by every integer except zero itself). thus 0<2−y0<2−y. statements together tell u Q-25Manhattan CAT2 DS (2) |3 – y| = 11 C A certain square is to be drawn on a coordinate plane. (B) that suggests that the unusually large contributions to its accounts are derived from government kickbacks. then 3 – y = 11 y = -8 right side If 3 – y < 0. -The author does not make the presumption that informed career and business choices are available to everyone. the coffee store has become a daily hang-out for more and more teenagers. 97 B.1st Attempt B.. I want you to discourage teenagers from coming here and start cultivating a mo Store manager: Are you sure? On average.html Gmat Club PS (E) 11/30 D contains fewer than 5 decimal places = terminating decimal .html C Percent = ( change / Original ) * 100 Is abc divisible by 32? 0 is divisible by any number (1) a. b. If the circus were to raise the ticket price by 5% and sell 200 tickets. @ can be 0. or y<2y<2 (if yy is more than or equal to 2.com/forum/m23-184340. presumes. since P(A)=0. The reverse is also true: if a number has even powers of its prime factors then it's a perfect square. 2 B. together to get a 20 kg bar with the final copper to nickel ratio of 5:11. If Bob tries to find the number by guessing the remaining digits at random.html E READ CAREFULLY . GMAT Club Tests DS (2) n is divisible by 19. try next largest sum. 2.29<0. say Copper (or Nickel if you fancy it more. d cannot be more than Event A (or B) happen individually The numbers {1. 1. or 1<x<31<x<3. He does not make any claim as to the r (D) demonstrating that the average teenage customer is as profitable as the average adult customer V-22 Manhattan CAT2 CR (E) offering new evidence refuting that presented by the owner (E) The new evidence that the manager presents neither contradicts nor refutes any of the evidence that the owner had previous presented. 3. The prosecutors have failed . No CR E. Not sufficient.and theEvent BB willthat probability happen bothbe more than individual p happen GMAT Club Tests DS (2) Probability that BB will NOT happen is 0. SC has raised fears that it is an attempt has raised fears that it is an attempt ---> it correctly refers to "Push" d then the sentence is wrong Some people believe that gaining wealth is due to luck. If they sit at tables with 6 members at each table except one and fewer than 6 members at that one table. If they meet 24 miles from point P. what is the largest possible value of pp? A.e: there are 2 ways he can have red and white marble (1) y ≤ 8 RED and WHITE Q-12Manhattan CAT2 DS (2) y ≥ 4 http://gmatclub. C. we have that x+y−3=−(1−y)x+y−3=−(1−y).html E Think of multiplies of 11 including 11*12 The product of three distinct positive integers is equal to the square of the largest of the three numbers.–Therefore. 120 GMAT Club Tests PS E. C Club X has more than 10 but fewer than 40 members. has raised fears that it is their attempt has raised fears that it is their attempt It / they must refer ONE thing in the sentence. is an integer that is the square of an integer. the weight of the first bar was 14 kg and the weight of the second bar was 6 kg. hence. Let's : 2!=2. thus x=2=primex=2=prime. Second bar is 3/8th copper and the combined alloy is 5/16th copper. 3| x 2 – 4| must b (2) INSUFFICIENT: To solve this equation for y. GMAT Club Tests PS E.com/forum/m25-184435. Not sufficient. This answer choice indicates that a third element could cause both the conditions described in the argument. Given that x=14p+2x=14p+2. which implies that −1<x−2<1−1<x−2<1. By asserting that the average teenager spends just as much money as the average adult and that there are more new customers than former customers. Two uses of that are probably not correct. Therefore. and the square is to have an area of 100. 4/625 2) = 1/25 D. don't forget the multiply with number of ways. d[x] denotes its smallest positive odd divisor and D[x] denotes its largest odd divisor.. have raised fears that it is an attempt have raised fears that it is an attempt have raised fears that they are trying have raised fears that they are trying has raised fears that they are trying has raised fears that they are trying --> in ithe above sentence "Push" is the subject . http://gmatclub. Forget about the other one. GMAT Club Tests DS (2) The largest number of the three distinct numbers is 24.thus they cannot refer to it and the remaining plural nouns are countries and emmisions thus there is an ambiguity .nor its officers guilty [not parallel] B) parallelism error: 1 ) that suggests that the unusually large contributions to its accounts are derived from government kickbacks" is a that clause. Sufficient. Sufficient. switch digits in 76 and we'll get: 67+73+71=211=pr Is x the square of an integer? The question basically asks whether xx is a perfect square (a perfect square. (A) 1 kg w1/w2 = (A2 – Aavg)/(Aavg – A1) (B) 4 kg w1/w2 = (3/8 – 5/16)/(5/16 – 2/7) = 7/3 (C) 6 kg (D) 14 kg So weight of first bar: weight of second bar = 7:3 om/blog/2011/04/quarter-wit-quarter-wisdom-dont-get-mixed-up-in-mixtures/ Veritas PS (E) 16 kg Out of a total of 20 kgs. and we have far more new customers than we have lost over the(A)past Theyear.7/0 UNDEFINED) (1) @! is not divisible by 5 (2) @@ is divisible by 9. Sufficient. If both bars are melted First bar is 2/7th copper (2 parts copper and 5 parts nickel to get a total of 7 parts). nor were its officers guilty 2)nor were its officers guilty d The General Agreement on Trade & Tariffs was a landmark deal in committing at its signing in 1973 to reduce trade restrictions around the world. we cannot determine the value of y. (2) When xx is divided by 14 the remainder is 2. thinking of positive can be enough for If 3 – y > 0. One bar has 2 parts of copper to 5 parts of nickel. overlooks the possibility that the same thing may causally contribute both to education and to gaining wealth.3. without providing justification that only highly educated people make informed career and business choices. Given that x=12q+6=6(2q+1)=2∗3∗(2q+1)x=12q+6=6(2q+1)=2∗3∗(2q+1). D. -Does the argument presume that to make an informed lifestyle choice a person must be highly educated? The author certainly believes that high educational levels lead to informed career and business choices. Since yy is a positive integer. Now. GMAT Club Tests DS (2) @ is divisible by 9 (1)+(2) Intersection of the values for @@ from (1) and (2) is @=0@=0. 2 mustonly be greater 14 is greater than or than equal or equal to 0. in the question above there are two elements in each mixture too (o There are 2 bars of copper-nickel alloy. So. He also remembers that the remaining digits are not 0. in committing at its signing in 1973 to reduce by committing at its signing in 1973 on the reduction of at its 1973 signing for committing to reduce for the commitment at its 1973 signing to reduce SC when it was signed in 1973 for its commitment to reduce d The push by rich countries to cut emissions have raised fears that it is an attempt to secure new markets for environmental technologies.html C 0/ 32 >>> divisible by 32 If a and b are two-digit positive integers greater than 10. 209 0!=1. is x a prime number? So. So the question asks whether @@ equals to 0 or 5. 7. what is the probability that the average daily deposit up to and including that day contains fewer than 5 decimal places? (A) 1/10 (B) 2/15 (C) 4/15 (D) 3/10 m/forum/every-day-a-certain-bank-calculates-its-average-daily-97456. xx could be 2. is N a multiple of 5? 1234@1234@ to be divisible by 5. If the sum of these three numbers is a prime number pp. 30. is x even? (1) D[x]−d[x]=0 GMAT Club Tests DS (2) D[3x]=3 http://gmatclub. or 4 (note that 0!=1).com/forum/m30-184576. R # Source Ty Question Comments OA Take Aways The core sentence is : The Federal investigators at Stapleton Industries have failed to find any evidence for 2 things : a)the unusually large contributions to its accounts are derived from government kickbacks b)its officers are guilty A has parallelism error and verb tense error: has suggested is incorrect. If a circus were to sell all of its 220 tickets for this month's performance at its usual price. does not acknowledge that some people who fail to make informed career and business choices have gained wealth. If there are several possibilities. what is the product of the two smaller numbers? (1) The average (arithmetic mean) of the three numbers is 34/3. let’s calculate the ratio of weights of the two bars. 100/21 D. C and E choices right away. 24/625 + 1/25 = 49/625 ~ 50/625 E Every day a certain bank calculates its average daily deposit for that calendar month up to and including that day. -The author specifically notes that gaining wealth is largely the result of making informed career and business choices.2nd. which in turn means (1) 3| x 2 – 4| = y – 2 (1) AND (2) SUFFICIENT: Statement (1) tells us that y is greater than or equal to 2. from government kickbacks. What was the weight of the first bar? Now. d Onlysoprime not afactorial prime. D.25<0. thus |1−y|=−(1−y)|1−y|=−(1−y). (1) @!@! is not divisible by 5. 5. then y−2≤0y−2≤0 and it cannot be greater than |x−2||x−2|) If x and y are positive integers. nor its officers guilty Screws up the original meaning by using the -Ing modifier. 16. Bob remembers the area code and the next 5 digits of the number. Perfect square always has even powers of its prime factors. what is the distance between the two cities? A. Though if you find that # of squares should be multiple of 4 you'll be left with A. with its officers guilty (A) that has suggested that the unusually large contributions to its accounts are derived from government kickbacks.com/forum/m15-184062. is it more likely that she will have 2 red marbles than that she will have one marble of each color? While calculating probability.0/0 NO . When the manager asks “Are you sure?” he is questioning the owner’s directive (or its effectiveness). GMAT Club Tests DS (2) The remainder when bb is divided by 12 is b http://gmatclub.71. -Correct. equals to the product of their individual probability.29.4. that informed career and business choices are available to everyone. you will get the same answer). The other has 3 parts of copper to 5 parts of nickel. including a 3-digit area code. each teenager spends just as much as the average adult does. 2)its officers guilty .Tells odd Is the product of two positive numbers x and y larger than their sum? (1) x and y are integers GMAT Club Tests DS (2) y>x>1 C Think of integers and non-integers always If nn is a positive integer is n"2−n divisible by 12 ? (1) n is divisible by 11. nor is needed because the argument focuses on a correlation betwee C. Therefore. http://gmatclub. Thus. powers of prime factors 2 and 3 are even (1) When x is divided by 12 the remainder is 6. Sufficient. Since my goal is to maximize our revenue. C 0!=1 Notice that since events AA and BB are independent. Of the two possible that y –values. thus xx cannot be a perfect squar GMAT Club Tests DS (2) When x is divided by 14 the remainder is 2. If you notice. 2 > 0. then 3 – y = -11 y = 14 What is the value of y? Since there are two possible values for y. If all coordinates of the vertices must be integers. and that statement indirectly allows other causes to lead D to the effect. 50/625 Add 1 and 2. xx may or may not be A a perfect square.25 (1) Probability that AA will happen is 0. 219 What is the largest possible sum of these three numbers that we can form? Maximize the first digit: 76+73+71=220=even76+73+71=220=even. and statement (2) tells us that y = -8 or 14. 7} are used to form three 2-digit numbers. Thus research supports the view that gaining wealth is largely the result of making informed career and business choices. research from many institutions worldwide indicates a strong correlation between gaining wealth and high educational levels. 9/4 GMAT Club Tests PS http://gmatclub. 192 e If N=1234@ and @ represents the units digit of N. The Federal investigators at Stapleton Industries have failed to find any evidence that has suggested that the unusually large contributions to its accounts are derived from C)suggesting government that kickbacks.com/forum/a-jar-contains-8-red-marbles-and-y-white-marbles-if-joan-101748. Sufficient. Either way. symbol "@@" should represent either 0 or 5. the account balance was 600 and for how D many was it 300.300). and solving the second equation gives us x = 800.html Gmat Club DS (2) (J+B)^2=400 http://gmatclub. method is to work backward from the answer choices and calculate how much Plans A and B pay out in each case.. 52. over. . 21 C. 34. The easiest way to calculate this probability is to use the “1-x shortcut. Sufficient. how many years will it A . However. First we need to determine how much Insurance Plan A pays. whichever is lower. 241.5{salary}={commission} --> {salary}>{commission}. but we don't know Or: which one. Stmnt 2 .com/forum/on-jane-s-credit-card-account-the-average-daily-balance-for-106680.html Gmat Club PS E. {commission}-{salary}=0. did the dealership have more than 100 vehicles in inventory at the beginning of the day Tuesday? From F. The answer is the c=600 and hence we need to use this value to work When Mrs. {salary}-{commission}=0. you have had less than 1/2 of the meal but certainly more than 1/4 of the meal. We are looking for the cost level for which both plans pay out the same amount. and more than half of the part-time employees of Company Z are employees Thus both statements individually are sufficient to solve the problem. thus the answer would be YES.5{salary}. right? If you eat only 1/4 of the sandwich and drink 1/4 of the milkshake. cannot be negative. 115.. So either X>Z>Y or ach-employee-of-company-z-is-an-employee-of-either-division-54083. Hence. R # Source Ty Question Comments OA Take Aways A woman sold 100 oranges at $12. 10. Max value of 17y=15317y=153 B S=10x+y (as max value of yy is 9). Was the bonus given greater than 17% Bonus .. the salesman's income would have been 10 percent higher last year --> 1. for rest of the days. and her monthly income has two components. C Understand the meaning If x2 < x and x is written as a terminating decimal. we know a>b of full-time employees to the number of part-time employees greater for division X than for Company Z? and d>c . From F.000 or 50% of total cost.5x $5. a+b/ c+d >b/d or. When the two calculated amounts are the sam Health insurance Plan A requires the insured to pay $1. does x have a nonzero hundredths digit? Think of all variations (1) 10x is not an integer. Say a meal consists of a sandwich and a milkshake. They begin at noon together. Answer: A. If J and B are both even numbers is J=B? they painted the wall in 288 minutes or in 288/60 hours. The last test Mrs. We know that # of Sedans sold on Tuesday = 85% of S = 1720∗S1720∗S = The b minimum value can be when S=20. the # of sedans sold on Tuesday = 17. Hence sufficient. i.html?kudos=1 Gmat Club DS (2) The dealership sold 85% as many sedans on Tuesday as it did on Wednesday. she awards a bonus.html?kudos=1 each of the 30 days. . If th 80% of the cost over $300. is a/c > (a+b)/(c+d) ? or. 38. and a variable component of $C for each set of encyclopaedias 1. %Full-Z will be a weighted average of %Full-X and %Full-Y. is the ratio of the number st(2). that {salary}-{comm A certain salesman's yearly income is determined by a base salary plus a commission on the sales he makes during the year. 25 Manhattan CAT3 PS $6. Did the salesman's base salary account for more than half of the salesman's yearly income last year? A. A person inherited few gold coins from his father.3 green balls means 2 WB 0= 3 + 2 = 5 balls in total 24/91 5!/5! = 1 45/91 . However. T scored was between 10 and 99.1({salary}+{commission})={salary}+1. How many were sold at the first rate? A. c=63k+52c=63k+52 means that # of coins can be: 52.. not possible. What is the number of coins he inherited from his father. the # of sedans sold at the end of the day = 89∗9k89∗9k = 8k = This represents 16th16thof the total sedans present at the start of the day. ad>bc.5{salary} --> 1.01%? A. What is the minimum value of x for which the probability that he flips all heads is less than 0. st(1). 25. (1) Jane's payment was credited on the 21st day of the billing cycle. Two of them are above the sales target in which case 2c=600 implying c=300 3. the dealership had sold 8/9 as many sedans as SUVs. is r > t ? (1) t > s 16 Manhattan CAT3 DS (2) r > s D Think simple.S=10x+y (as the score was between 10 and 99). The rest of the 5 days. D . some at the rate of 3 for 35 cents and the rest at 7 for 85 cents. $1. where n>0. All three of them are above the sales target in which case 3c=600 implying c=200 that she sells in that month over a sales target of n sets. Thus.5{salary}={commission} --> {salary}<{commission}.ab 25 Manhattan CAT3 DS (2) 100x is an integer.8(x – 300) = . Thus.Multiply them to get the # of ways having 3G + 2 W balls 28 Manhattan CAT3 PS 84/91 Therefore. so st(2) alone is sufficient too.” The only way the jury will have fewer than 8 men is if a jury of 7 men and 5 women (the maximum number of women available) is selected. General formula for cc based on above two statements will be: c=63k+52 (the divisor should be the least common multiple of above two divisors 9 and 7. All of the women would have to be on the jury. forum/jane-can-paint-the-wall-in-j-hours-and-bill-can-paint-the-87131. B On Jane's credit card account. Plan B requires the insured to pay the initial $300. one is already proved to be wrong. what is the probability that there will be 3 green balls the jury will comprise at least 2/3 men? 10! / 7!3! = 120 The number of ways we could select 5 women from a pool of 5 women is: . C 0. 0. and Bill can paint the same wall in B hours. We need to find the average of the amount in average daily balance on Jane's account for the billing cycle? Stmnt 1 ..html?kudos=1 Gmat Club DS (2) Piglet A was fed exactly 1/6 of the barley today This 1/n must lie between 1/4 and 1/6. meaning that we can have two cases: (2) The difference between the amount of the salesman's base salary and the amount of the commission was equal to 50 percent of the salesman's base salary last A.html Gmat Club DS (2) If the herd were to grow in number at twice its current rate. T added 16 bonus points to the last test she graded --> B=2x=16B=2x=16 --> x=8x=8 --> 30x=24030x=240. how many piglets are there in the litter? Since it is given that each piglet gets the same amount of food and there are n Another thing to consider here is that number of piglets has to be a positive integer. If each division has some part time employees. or at least 8 men (2/3 of 12 jurors = 8 men).Calculate can happen. Can I say you have had 1/2 of the meal? Sure.000 Because there are two possible payment structures for Plan A. 12 C Check answers Randy is going to flip a coin x times. 178. the answer has to be one out of 2 and 3. it is fed less than 1/4 of the total food but more than 1/6 of the total food. 15 orum/a-woman-sold-100-oranges-at-12-10-some-at-the-rate-of-98032.000 or 50% of the total cost. If no other amounts were added to or subtracted from the account during the billing cycle. X=a+b Multiplying the both equation gives us. %20 increases means A* (1+0. 10 B. . If he put 9 coins in each bag then 7 coins are left over. and there is only one way that .html Gmat Club DS year. Which of the following is a cost level for which both insurance plans pay out the same amount? $600 For Insurance Plan B. so # of coins can be: 7. but So when piglet A is fed 1/4 of the Oats and 1/6 of the Barley. So Insurance Plan B pays 0.2) take for the number of antelope to double? Growth at some rate means that we have exponential growth. You can easily find the average. My solution: (From the diagram table) we have to know. her income for that month would have been over $4000. m/each-piglet-in-a-litter-is-fed-exactly-one-half-pound-of-a-82321. Laura sells encyclopaedias. absolute value sign ||.Calculate choosing 3 out of 5 green balls 2/3 This makes practical sense. The two $3. (2) Mrs. there would be 980 antelope in the group in two years. However. A car dealership carries only sedans and SUVs. (1) By the end of the day.m. the ratio automatically decreases in division Y to keep the weighted Z = (a+c) average 4 ( b+(ratio) d) ---->>> samea+c for/ company b+d mustZ. but not both.We know now that the account had $600 for 20 days and $300 for the rest of the 10 days.html A = In one hour. and the insurance plan pays 80% of everything over $300. So. whichever is lower.The rest is classical probability Jane can paint the wall in J hours. 45. Did the salesman's base salary account for more than half of the salesman's yearly income last year? Given: {Income}={salary}+{commission}. they painted 60/288 of the wall. as the # of cars sold can only assume integral values. but then pays An algebraic approach is the other possibility. while evaluating the first statement you got c=200 because you considered all three sets of encyclopedias to be above the sales target of n sets.html Gmat Club PS E.5{salary} --> 0.beThis somewher reasoning between is for Statement a/b and c/d.S 2. then you would have had only 1/4 of the meal. (1) The base score of the test was between 50 and 90 --> 5≤x≤9 --> the range of 30x is 150≤30x≤270 and the range of 17y is 0≤17y≤153 (as yy can be from 0 to 9). so we need to set up 2 equations. For some days the account had $600. the # of sedans at the start of the day = 8k*6 = 48k. Though try to identify that the question is just testing you on weighted averages. However if he puts 7 coins in each bag then 3 coins are left If he puts 7 coins in each bag then 3 coins are left over --> c=7p+3. so # of coins can be: 3. Not sufficient. A* x (1) Ten years from now. then Avg = (600*20 + 300*10)/30 = 500 Gmat Club DS (2) The average daily balance through the 25th day of the billing cycle was $540. If there are 500 antelope in the herd today. but $3. ad+bd>bc+bd a>b or. Jane's credit card account had a balance of $600. = |x-y| cuse we dont know which one notice is bigger. T's students answer the bonus question correctly. hence 30x30x is more than 17y17y.. You don't need to since it is a DS question.. Hence n must be 5. the bonus is equal to 2 times the tens digit Given: Score is a two digit number .com/forum/jane-can-paint-the-wall-in-j-hours-and-bill-can-paint-the-87131. the average daily balance for 30-day billing cycle is the average (arithmetic mean) of the daily balances at the end of http://gmatclub. So piglet A (1) Piglet A was fed exactly 1/4 of the oats today each piglet gets 1/n of the total food.html Gmat Club DS (2) If he put 13 coins in each bag then no coin is left over and number of coins being lesser than 200. say 'n'.800 0. since it is given that each piglet gets the same piglets. 13 randy-is-going-to-flip-a-coin-x-times-what-is-the-minimum-value-of-x-194527. what was the Long question stem. 17. (1) Jane and Bill finish at 4:48 p. Let x be the total cost. Z will always be in the middle. amount of food and there are n piglets. so we can set the two plans equal to each other. 59. and the student answered the bonus question correctly. the probability that the jury will be comprised of at least 8 men is 1 – (24/91) = 67/91. 52. 31. what this n Secondly. Only one of them is above the sales target in which case c=600 (1) If Laura had sold three fewer sets in March. 2) (2) More than half of the full-time employees of Company Z are employees of Division X. there are three possibilities.3{commission} --> {salary}=2{commission} --> {salary}>{comm (2) The difference between the amount of the salesman's base salary and the amount of the commission was equal to 50 percent of the salesman's The difference base salary between last year X and -->Y|{salary}-{commission}|=0. A B Y-X when you combine both the statements together. (1) If the amount of the commission had been 30 percent higher. in addition to mathematical sense. and the jury pool consists of 2/3 men and 1/3 women. forum/a-person-inherited-few-gold-coins-from-his-father-if-he-132500. At the beginning of a certain 30-day billing cycle.000 OR 0. We can approach this problem in two ways. and probably easier. If he puts 9 coins in each bag then 7 coins are left over --> c=9q+7.300 Solving the first equation gives us x = 3. combine in equalities. The first. You eat 1/2 of the sandwich and drink 1/2 of the milkshake. choosing Putting2 out these of selections 8 white balls together. 9 D.8(x – 300) = x – 1. The question asks for the probability that the jury will comprise at least 2/3 men. How much did she earn in March? 2. Substituting c=300 in the statement 2 gives us a Cvalue of n=0. ad>bc ? Inequalities can be multiplied. each piglet will get 1/n of the total food.html?kudos=1 Gmat Club DS (2) Mrs. so 63 and the remainder should be the first common integer in above two pat (1) The number of coins lies between 50 to 120. 61. Y=c+d (1) the ratio of the number of full time employees to the number of part-time employees is less for division Y than for Company Z In the weighted average if ratio (full time / part time) increases in division X. as this is 16th16th rate = 1 + x he number of antelope in a certain herd increases every year at a constant rate. in the base score. what is the probability that The number of ways we could select 7 men from a pool of 10 men is: we will choose 5 balls. laura-sells-encyclopaedias-and-her-monthly-income-has-two-64646.200 $800 isn't one of our options. T added 16 bonus points to the last test she graded. 11 C. Now. we know that n denotes the number of set of books and thus. but if you had to find the average. 12 D. and the only change in inventory was that some vehicles were sold. you get the value of n t be less than -5. 45 B. 16. /if-the-base-score-is-between-10-and-99-the-bonus-is-equal-95183.e. 24. d>c a*d > b*c Each employee of Company Z is an employee of either division X or division Y.800 is. The no of SUV's sold can be minim forum/a-car-dealership-carries-only-sedans-and-suvs-159268. the insured pays the first $300. Question basically asks: is {salary}>{commission}? (1) If the amount of the commission had been 30 percent higher.000"). 43. Only 1/5 lies between 1/4 and 1/6 (such that n is a positive integer). there will be more than ten times the current number of antelope in the herd. b 1. the account had $300 in it. Also.8(x . If the base score is between 10 and 99. Sufficient. Not sufficient. Hence 30x may or may not be more than 17y. the answer does not stand. we know that the # of SUV's sold at the end of the day were of the form 9k(k is a a positive integer).Average for first 25 days = 540 so you can find for how many of these 25 days. The ratio of the amount of barley to that of oats varies from piglet to piglet. .B=2x of the base score? Question: is 2x>17/100(10x+y) ? --> is 30x>17y when it is about digits write down the number as: (1) The base score of the test was between 50 and 90. thus the answer would be No. right? Each piglet in a litter is fed exactly one-half pound of a mixture of oats and barley. The word "difference" implies that one of them IS bigger. the salesman's income would have been 10 percent higher last year.. number of piglets = n each piglet is fed some of both grains. the balance was of course $300. Let the # of sedans sold on Wednesday be S. working together. A. a fixed component of $1000. Since.abc 12 people will be selected from a pool of 15 people: 10 men (2/3 of 15) and 5 women (1/3 of 15). A + x . X-Y um/a-certain-salesman-s-yearly-income-is-determined-by-a-base-126533. C A certain salesman's yearly income is determined by a base salary plus a commission on the sales he makes during the year. What happens in case you eat 1/2 of the sandwich but drink only 1/4 of the milkshake? In that case. 14 E If r + s > 2 t. and on Tuesday it sold 1/6 of the sedans that it had in stock at the beginning of the day. the number of ways a jury of 7 men and 5 wom 67/91 The probability that the jury will be comprised of fewer than 8 men is thus 120/455 = 24/91. Growth at some constant amount means linear growth (for example if we were told that "the number of antelope in a certain herd increases every year by orum/the-number-of-antelope-in-a-certain-herd-increases-every-99810.html?kudos=1 Gmat Club DS of Division Y Thus Answer is (D) D X<Z<Y. her income for that month would have been $600 lower than it was.S 1. If no new inventory arrived at any point on Tuesday. There cannot be fewer The total number of juries that could be randomly selected from this jury pool is: 5 Green 8 white balls 15! / 12!3! = 455 If a jury of 12 people is to be selected randomly from a pool of 15 potential jurors.800.html?kudos=1 Gmat Club DS (2) If Laura had sold 10 sets of encyclopaedias in March. Jane made a payment of $300 on the account during the billing cycle.10.a 0. ad>bc so st(1) alone is sufficient. ac+ad>ac+bc or. The question states that the insured pays either $1.
Copyright © 2024 DOKUMEN.SITE Inc.